Semiklassischer Grenzwert der Quantenmechanik

Ich finde mich oft verwirrt über die unterschiedlichen Definitionen, die man im Kontext der Quantenmechanik " semiklassischen Grenzen " gibt, mit anderen Worten Grenzen, die die Quantenmechanik schließlich zur klassischen Mechanik machen.


In handgewellter Manier

  • Die klassische oder halbklassische Grenze entspricht der Einnahmegrenze 0.
  • Wenn man vom Korrespondenzprinzip spricht, erhält man oft den semiklassischen Limes im Limes großer Quantenzahlen (große Bahnen und Energien).

Etwas präziser

  • Exemplarische Quelle der Verwirrung: Eine Möglichkeit, zu zeigen, warum 0 eine klassische Grenze beschreibt, lautet wie folgt:

Nehmen Sie die 1 D Schrödinger-Gleichung für ein Massenteilchen m in einem Potential v ( x ) :

ich t ψ ( x , t ) = [ 2 2 m 2 + v ( x ) ] ψ ( x , t )

Durch Einfügen ψ ( x , t ) = e ich S ( x , t ) / in der Schrödinger-Gleichung oben und vereinfachend für ψ , wir erhalten:

S t = 1 2 m ( S ) 2 ich 2 m ( 2 S ) + v
Jetzt nehmen 0 , das Obige wird einfach zur klassisch bekannten Hamilton-Jacobi-Gleichung , wobei S beschreibt Hamiltons Hauptfunktion oder die Aktion:

S t = 1 2 m ( S ) 2 + v
Wenn wir ein solches Ergebnis verwenden, können wir an verwenden Erweiterung von S in der zweiten Gleichung. Leider verstehe ich nicht, warum das Erreichen der Hamilton-Jacobi-Gleichung notwendigerweise ein klassisches Verhalten impliziert!

  1. Alternativ spricht man von klassischen Grenzen der QM, indem man sagt: Beim Planckschen Quant sehr klein im Vergleich zu Werten des Lagrangeschen Wirkungsintegrals ( Feynmans Pfadintegralformalismus ). Ich sollte das wahrscheinlich nicht fragen (da die Diskussion hier ziemlich vage ist), aber gibt es eine nette Möglichkeit, die obige Idee mathematisch zu demonstrieren? (z. B. indem gezeigt wird, ob eine solche Grenze notwendigerweise zu Quantendekohärenz führt und daher die klassischen Trajektorien dominant werden.)

  2. Schließlich sind die beiden Aussagen von 0 und unter der Grenze von hohen Quantenzahlen irgendwie äquivalent? (dh eine Neuformulierung voneinander?)

Natürlich sind auch alle anderen Möglichkeiten (ob physikalisch oder mathematisch) des Nachdenkens und Verstehens halbklassischer Grenzen der Quantenmechanik als Antwort willkommen.

Mögliche Duplikate: physical.stackexchange.com/q/17651/2451 , Physics.stackexchange.com /q/32112/2451 , Physics.StackExchange.com/q/33767/2451 , Physics.StackExchange.com/q/ 56151 / 2451 und Links darin.
@Phonon wir können die Winger-Funktion verwenden und die Wellenfunktion selbst in der klassischen Grenze interpretieren. Dies zu tun und das Integral zu manipulieren würde die Phasenraumgleichungen unter der Bedingung ergeben, dass 0

Antworten (2)

Erstens sind die klassischen und halbklassischen Adjektive nicht ganz synonym. "Semiklassisch" bedeutet eine Behandlung eines Quantensystems, dessen Teil klassisch und ein anderer Teil quantenmechanisch beschrieben wird. Felder können klassisch sein, Teilchenpositionen innerhalb der Felder quantenmechanisch; Metrische Felder können klassisch sein und andere Materiefelder sind quantenmechanisch und so weiter.

Außerdem behandeln wir den „Quantenteil“ der semiklassischen Behandlung oft in einer anderen Annäherung – wo wir nur das führende klassische Verhalten plus die erste Quantenkorrektur nehmen. Für diesen Teil des Systems bedeutet „semiklassisch“ also „Einschleifen-Approximation“ (wie in der WKB-Approximation).

Nun kann gezeigt werden, dass die Gesetze der Quantenmechanik die Gesetze der klassischen Physik für alle "klassischen Fragen" wann immer implizieren 0 . Genauer gesagt, 0 tatsächlich bedeutet J / für alle normalen "Winkelimpulse" J , Aktionen S (Anstatt von J ) und alles andere mit den gleichen Einheiten. Also ja, in der Tat, die 0 der klassische Grenzwert und der Grenzwert der großen Quantenzahlen ist dasselbe. Es ist nicht koscher zu fragen, ob eine große Menge wie z ist viel kleiner als eins; ob der Zahlenwert klein ist, hängt von den Einheiten ab. Also müssen wir diese Aussagen über „sehr klein“ oder „sehr groß“ dimensionslos machen, und deshalb brauchen wir das eben nicht aber auch J oder S des eigentlichen Problems, und deshalb sind alle Ungleichungen, die die von Ihnen erwähnte klassische Grenze diktieren, äquivalent.

In dieser Grenze werden die Spektren so dicht, dass die Observablen (wie die Energie des Wasserstoffatoms) effektiv kontinuierlich sind, obwohl sie in der exakten Quantenbehandlung diskret sind. Die Heisenberg-Bewegungsgleichungen für die Operatoren reduzieren sich auf die klassischen Bewegungsgleichungen. Die Dekohärenz garantiert, dass bei einigen Umgebungen die diagonalen Einträge der Dichtematrix als klassische Wahrscheinlichkeiten interpretiert werden können und die nicht-diagonalen schnell auf Null gehen. Wir können uns immer vorstellen, dass die Wellenfunktionen in diesem Grenzfall "enge Pakete" sind, deren Breite vernachlässigbar ist und deren Mittelpunkt sich gemäß den klassischen Gleichungen bewegt. Es funktioniert einfach.

Man sollte alle Aspekte dieses Beweises verstehen, dass „die klassische Physik eine Grenze der Quantenmechanik ist“, was davon ausgeht, wie wir die Fragen stellen und sie von einem Formalismus in einen anderen übersetzen sollten, und so weiter. Aber am Ende ist die Tatsache, dass diese Aussage gilt, wichtiger als einige technische Details des Beweises.

Historisch gesehen ist die Hamilton-Jacobi-Gleichung eine Möglichkeit, die klassische Physik zu beschreiben, da sie lange vor der ersten Begegnung mit der Quantentheorie entdeckt und als gleichwertig mit der klassischen Physik gezeigt wurde. Mathematisch können Sie sehen, dass die Hamilton-Jacobi-Gleichung nur die Größen enthält, die wir tatsächlich mit klassischen Geräten wie messen können S , t , v , m usw. und es hängt nicht davon ab überhaupt – auch wenn man zum Beispiel die SI-Einheiten verwendet – was beweist, dass die Gleichung unabhängig von der Quantenmechanik ist.

Es gibt viele Dinge über die klassische Grenze der Quantenmechanik und einige spezifischere Klassen quantenmechanischer Theorien zu sagen, siehe z

http://motls.blogspot.com/2011/11/how-classical-fields-particles-emerge.html?m=1

So gut geschrieben und beantwortet ... +1

Es gibt Ergebnisse, die mathematisch streng sind, was den semiklassischen Grenzwert von Quantentheorien betrifft. Es ist in der Tat ein andauerndes und interessantes Thema der Forschung in der mathematischen Physik. Sie müssen jedoch ziemlich gut in der Analyse versiert sein, um die Ergebnisse zu verstehen. Die Bibliographie ist ziemlich umfangreich, aber ich möchte die folgenden (teils ziemlich alten) Ergebnisse erwähnen:

Endlich dimensionaler Phasenraum (Quantenmechanik):

  • Hepp 1974 Methode der kohärenten Zustände.

  • Helffer, Martinez und Robert 1987, auf Französisch . Verwendet den sogenannten Wigner-Measure-Ansatz.

  • Figalli, Ligabo, Paul 2010 . Moderne Herangehensweise an Wigner-Maßnahmen, Umgang mit groben Potenzialen.

Unendlicher dimensionaler Phasenraum (bosonische QFT)

  • Ginibre und Velo 1979 Erweiterung des Werks von Hepp ins Unendliche.

  • Ammari und Nier 2007 Unendliche dimensionale Wigner-Maße.

  • BBGKY-Hierarchie: Überprüfung durch Golse (mittlere Feldgrenze, die mathematisch der semiklassischen Grenze entspricht)

Auch diese Folien von Francis Nier können sich für einen schnellen Überblick über endlich- und unendlichdimensionale Wigner-Maße als nützlich erweisen.

Ich werde nicht versuchen, die Ideen zu erklären, weil es sehr technisch und sehr lang wäre. Ohne genauer zu sein, kann ich Ihnen sagen, dass sie die Grenzen rigoros untersuchen 0 (oder äquivalent auch wenn N , wo N ist die Anzahl der Teilchen im System), um zu beweisen, dass sich die lineare unitäre Quantendynamik im Grenzfall auf die nichtlineare klassische Dynamik reduziert. Tut mir leid, aber ich habe keine Zeit, mehr als das zu sagen ;-)